Knapsack Problem with Equal Weights











up vote
0
down vote

favorite












The problem consists in the standard knapsack problem in interger programming with the weights that all have the same values, for example they are all equal to one.



It seems to me that the solution of this problem should be trivial, that is first set to one the variable with the highest profit, then the one with the second highest profit and so on, until the knapsack is full.



I was wondering if there is a formal way to prove that this procedure reaches a global minimum, or if there are papers that deal with this particular instance of the knapsack problem.



Thank you.










share|cite|improve this question


























    up vote
    0
    down vote

    favorite












    The problem consists in the standard knapsack problem in interger programming with the weights that all have the same values, for example they are all equal to one.



    It seems to me that the solution of this problem should be trivial, that is first set to one the variable with the highest profit, then the one with the second highest profit and so on, until the knapsack is full.



    I was wondering if there is a formal way to prove that this procedure reaches a global minimum, or if there are papers that deal with this particular instance of the knapsack problem.



    Thank you.










    share|cite|improve this question
























      up vote
      0
      down vote

      favorite









      up vote
      0
      down vote

      favorite











      The problem consists in the standard knapsack problem in interger programming with the weights that all have the same values, for example they are all equal to one.



      It seems to me that the solution of this problem should be trivial, that is first set to one the variable with the highest profit, then the one with the second highest profit and so on, until the knapsack is full.



      I was wondering if there is a formal way to prove that this procedure reaches a global minimum, or if there are papers that deal with this particular instance of the knapsack problem.



      Thank you.










      share|cite|improve this question













      The problem consists in the standard knapsack problem in interger programming with the weights that all have the same values, for example they are all equal to one.



      It seems to me that the solution of this problem should be trivial, that is first set to one the variable with the highest profit, then the one with the second highest profit and so on, until the knapsack is full.



      I was wondering if there is a formal way to prove that this procedure reaches a global minimum, or if there are papers that deal with this particular instance of the knapsack problem.



      Thank you.







      optimization integer-programming






      share|cite|improve this question













      share|cite|improve this question











      share|cite|improve this question




      share|cite|improve this question










      asked Nov 13 at 22:42









      Vittorio Latorre

      82




      82






















          1 Answer
          1






          active

          oldest

          votes

















          up vote
          0
          down vote













          Try proof by contradiction. Assume all weights are 1 and the knapsack capacity is $C$. Posit an optimal solution that includes at least one item not among the $C$ most valuable items, and prove that you can improve it (contradicting optimality).






          share|cite|improve this answer





















            Your Answer





            StackExchange.ifUsing("editor", function () {
            return StackExchange.using("mathjaxEditing", function () {
            StackExchange.MarkdownEditor.creationCallbacks.add(function (editor, postfix) {
            StackExchange.mathjaxEditing.prepareWmdForMathJax(editor, postfix, [["$", "$"], ["\\(","\\)"]]);
            });
            });
            }, "mathjax-editing");

            StackExchange.ready(function() {
            var channelOptions = {
            tags: "".split(" "),
            id: "69"
            };
            initTagRenderer("".split(" "), "".split(" "), channelOptions);

            StackExchange.using("externalEditor", function() {
            // Have to fire editor after snippets, if snippets enabled
            if (StackExchange.settings.snippets.snippetsEnabled) {
            StackExchange.using("snippets", function() {
            createEditor();
            });
            }
            else {
            createEditor();
            }
            });

            function createEditor() {
            StackExchange.prepareEditor({
            heartbeatType: 'answer',
            convertImagesToLinks: true,
            noModals: true,
            showLowRepImageUploadWarning: true,
            reputationToPostImages: 10,
            bindNavPrevention: true,
            postfix: "",
            imageUploader: {
            brandingHtml: "Powered by u003ca class="icon-imgur-white" href="https://imgur.com/"u003eu003c/au003e",
            contentPolicyHtml: "User contributions licensed under u003ca href="https://creativecommons.org/licenses/by-sa/3.0/"u003ecc by-sa 3.0 with attribution requiredu003c/au003e u003ca href="https://stackoverflow.com/legal/content-policy"u003e(content policy)u003c/au003e",
            allowUrls: true
            },
            noCode: true, onDemand: true,
            discardSelector: ".discard-answer"
            ,immediatelyShowMarkdownHelp:true
            });


            }
            });














            draft saved

            draft discarded


















            StackExchange.ready(
            function () {
            StackExchange.openid.initPostLogin('.new-post-login', 'https%3a%2f%2fmath.stackexchange.com%2fquestions%2f2997459%2fknapsack-problem-with-equal-weights%23new-answer', 'question_page');
            }
            );

            Post as a guest















            Required, but never shown

























            1 Answer
            1






            active

            oldest

            votes








            1 Answer
            1






            active

            oldest

            votes









            active

            oldest

            votes






            active

            oldest

            votes








            up vote
            0
            down vote













            Try proof by contradiction. Assume all weights are 1 and the knapsack capacity is $C$. Posit an optimal solution that includes at least one item not among the $C$ most valuable items, and prove that you can improve it (contradicting optimality).






            share|cite|improve this answer

























              up vote
              0
              down vote













              Try proof by contradiction. Assume all weights are 1 and the knapsack capacity is $C$. Posit an optimal solution that includes at least one item not among the $C$ most valuable items, and prove that you can improve it (contradicting optimality).






              share|cite|improve this answer























                up vote
                0
                down vote










                up vote
                0
                down vote









                Try proof by contradiction. Assume all weights are 1 and the knapsack capacity is $C$. Posit an optimal solution that includes at least one item not among the $C$ most valuable items, and prove that you can improve it (contradicting optimality).






                share|cite|improve this answer












                Try proof by contradiction. Assume all weights are 1 and the knapsack capacity is $C$. Posit an optimal solution that includes at least one item not among the $C$ most valuable items, and prove that you can improve it (contradicting optimality).







                share|cite|improve this answer












                share|cite|improve this answer



                share|cite|improve this answer










                answered Nov 18 at 4:19









                prubin

                1,320125




                1,320125






























                    draft saved

                    draft discarded




















































                    Thanks for contributing an answer to Mathematics Stack Exchange!


                    • Please be sure to answer the question. Provide details and share your research!

                    But avoid



                    • Asking for help, clarification, or responding to other answers.

                    • Making statements based on opinion; back them up with references or personal experience.


                    Use MathJax to format equations. MathJax reference.


                    To learn more, see our tips on writing great answers.





                    Some of your past answers have not been well-received, and you're in danger of being blocked from answering.


                    Please pay close attention to the following guidance:


                    • Please be sure to answer the question. Provide details and share your research!

                    But avoid



                    • Asking for help, clarification, or responding to other answers.

                    • Making statements based on opinion; back them up with references or personal experience.


                    To learn more, see our tips on writing great answers.




                    draft saved


                    draft discarded














                    StackExchange.ready(
                    function () {
                    StackExchange.openid.initPostLogin('.new-post-login', 'https%3a%2f%2fmath.stackexchange.com%2fquestions%2f2997459%2fknapsack-problem-with-equal-weights%23new-answer', 'question_page');
                    }
                    );

                    Post as a guest















                    Required, but never shown





















































                    Required, but never shown














                    Required, but never shown












                    Required, but never shown







                    Required, but never shown

































                    Required, but never shown














                    Required, but never shown












                    Required, but never shown







                    Required, but never shown







                    Popular posts from this blog

                    QoS: MAC-Priority for clients behind a repeater

                    Ивакино (Тотемский район)

                    Can't locate Autom4te/ChannelDefs.pm in @INC (when it definitely is there)